LSAT and Law School Admissions Forum

Get expert LSAT preparation and law school admissions advice from PowerScore Test Preparation.

 Administrator
PowerScore Staff
  • PowerScore Staff
  • Posts: 8917
  • Joined: Feb 02, 2011
|
#47205
Please post your questions below!
 nancy_sweet
  • Posts: 1
  • Joined: Nov 06, 2018
|
#60177
Why is D not correct?
 mjb514
  • Posts: 35
  • Joined: Nov 01, 2017
|
#60288
I have the same question as above. I picked B because it seemed stronger, but D seems like it works as well.
 Brook Miscoski
PowerScore Staff
  • PowerScore Staff
  • Posts: 418
  • Joined: Sep 13, 2018
|
#60294
Hi, Nancy and mjb

The question stem asks you to justify the Environment Minister's argument.The "justify" task is to prove the argument true.

Answer choice (D) only says that economics are a "consideration." Answer choice (D) still allows economics to be outweighed by other considerations, such as pollution.

Answer choice (B) says that economics are more important than pollution, which offers much stronger support for the Environment Minister's argument, since he is arguing that the country should choose the economy over the environment.

Again, when you see a word like "justify" you are trying to pick a choice that is sufficient to make the argument true.
 cecilia
  • Posts: 66
  • Joined: Nov 07, 2011
|
#65498
—Hi - just following up on that one last line in Brooke's explanation. "Again, when you see a word like "justify" you are trying to pick a choice that is sufficient to make the argument true."

Just seeing the word "justify" , tho, is not enough to indicate it's always and forever a sufficient question, tho, correct???? For example, if the quesiton asks for "what most helps justify", that would be a Strengthen q, woudln't it???

Thanks in advance for any clarification!!!
User avatar
 KelseyWoods
PowerScore Staff
  • PowerScore Staff
  • Posts: 1079
  • Joined: Jun 26, 2013
|
#66893
Hi Cecilia!

You are correct, the word "justify" alone is not enough to determine that you are dealing with a Justify question stem. If the degree of justification is lessened in any way (usually by putting a "most" in front of "justify"), then you no longer have to find an answer that justifies or proves the argument, just one that strengthens it as much as possible. So you are correct that if you have a question stem which asks you which answer choice does the "most to justify," you should treat it as a Strengthen question.

Best,
Kelsey
User avatar
 fleurgirl
  • Posts: 3
  • Joined: May 29, 2021
|
#87795
Hello,

Why is E not the correct answer? Couldn't it be through conditional reasoning?

Thanks in advance!
 Robert Carroll
PowerScore Staff
  • PowerScore Staff
  • Posts: 1787
  • Joined: Dec 06, 2013
|
#87808
fleur,

Answer choice (E) is nearly a Mistaken Negation of what the argument wants. Look at the relation between the premises and the conclusion:

Premises: This would likely help the environment, but also likely reduce economic growth.

Conclusion: We shouldn't do it.

Answer choice (E) is a conditional:

(policy likely protects environment) + (policy unlikely to reduce economic growth) :arrow: implement policy

So if something is likely to protect the environment and unlikely to reduce economic growth, we should do it. But the argument in the stimulus wants us not to do something because it is likely to reduce economic growth. It's true that answer choice (E) matches the stimulus when it talks about the likelihood of the policy protecting the environment, but the policy is not unlikely to reduce economic growth. The conditional just does not apply. At best, that conditional tells me when to do something, not when to refrain from something.

Robert Carroll
User avatar
 KwakuS
  • Posts: 35
  • Joined: Jun 03, 2021
|
#96499
Would the contrapositive apply here? So that it would be that if a country decided not to implement a policy, it must have been because the policy either hurt the environment or was likely to reduce economic growth? That's how I initially saw the answer.

Thanks,
Kwaku
 Adam Tyson
PowerScore Staff
  • PowerScore Staff
  • Posts: 5153
  • Joined: Apr 14, 2011
|
#97239
I don't see this as being conditional, Kwaku, so there isn't a contrapositive that we could use.

Approach it this way:

Premise: The agreement would hurt economic growth while helping reduce pollution.

Conclusion: We shouldn't sign it.

What would help us get from that premise to that conclusion? Something that says "we should not do things that hurt economic growth, even if they reduce pollution." Answer B is a close match to that idea, linking the premise to the conclusion more securely. Close that gap in the argument in order to strengthen it.

Get the most out of your LSAT Prep Plus subscription.

Analyze and track your performance with our Testing and Analytics Package.